What is the Justification for Theorem XII in Taylor and Mann's Book?

  • Thread starter Thread starter Jamin2112
  • Start date Start date
  • Tags Tags
    Theorem
Click For Summary
The discussion centers on the justification for Theorem XII from Taylor and Mann's book, which states that the radius of convergence R of a power series is determined by the limit superior of the sequence of coefficients. The proof utilizes Cauchy's root test, establishing that the limit superior of the absolute values of the coefficients, |an|1/n, is essential for determining R. Concerns are raised about the implications of using the limit inferior instead, as it would yield a larger radius of convergence, which contradicts the goal of finding the largest R. The necessity of using limit superior is emphasized, as the Cauchy root test is specifically designed to work with it. Understanding this distinction is crucial for correctly applying the theorem in the context of power series convergence.
Jamin2112
Messages
973
Reaction score
12

Homework Statement



I'm having trouble understanding the justification for THEOREM XII in Taylor and Mann's book.

Homework Equations



THEOREM XII. The radius of convergence R of a power series ∑anxn is given by

1/R = lim sup |an|1/n.​

The Attempt at a Solution



Here is the proof which follows the theorem:

Proof. We appeal to Cauchy's root test. Let un = anxn.
Then
lim sup |un|1/n = |x| lim sup |an|1/n = |x| / R,​

where R is defined by R = 1 / (lim sup |an|1/n).

But why is it lim sup |an|1/n ? If you have another cluster point, say lim inf |an|1/n, then that will give us a larger R, since it will make the denominator larger in R = 1 / (lim sup |an|1/n). When we look for the radius of convergence, we look for the largest R, right?
 
Physics news on Phys.org
You apply the Cauchy root test. This test will only work with limsup. To see why it doesn't work with liminf, you'll need to check the proof of the Cauchy root test...
 
Question: A clock's minute hand has length 4 and its hour hand has length 3. What is the distance between the tips at the moment when it is increasing most rapidly?(Putnam Exam Question) Answer: Making assumption that both the hands moves at constant angular velocities, the answer is ## \sqrt{7} .## But don't you think this assumption is somewhat doubtful and wrong?

Similar threads

  • · Replies 2 ·
Replies
2
Views
2K
  • · Replies 1 ·
Replies
1
Views
2K
  • · Replies 3 ·
Replies
3
Views
1K
  • · Replies 7 ·
Replies
7
Views
3K
  • · Replies 4 ·
Replies
4
Views
2K
  • · Replies 3 ·
Replies
3
Views
1K
  • · Replies 7 ·
Replies
7
Views
2K
  • · Replies 2 ·
Replies
2
Views
2K
  • · Replies 3 ·
Replies
3
Views
2K
  • · Replies 2 ·
Replies
2
Views
3K